• We need your support!

    We are currently struggling to cover the operational costs of Xtremepapers, as a result we might have to shut this website down. Please donate if we have helped you and help make a difference in other students' lives!
    Click here to Donate Now (View Announcement)

As physics p1 MCQS YEARLY ONLY.

Messages
164
Reaction score
193
Points
53
Messages
8,477
Reaction score
34,837
Points
698
18)Efficiency is given by (Useful Output Work)/(Total Input Energy) =
(Useful Output Work per Second)/(Total Input Energy per Second)
The useful output work per second = useful power output = 150 * 10^3 Joules Per Second.
Therefore, Efficiency = (150 * 10^3 Joules/Second)/(Total Input Energy per second)
Every hour, 20 liters of fuel are consumed. Therefore, every second, 20/(60 * 60) liters of fuel are consumed.
The energy in these liters is equal to [20/(60 * 60)] * 40 * 10^6 Joules.
Therefore, the efficiency is equal to
(150 * 10^3)/[20/(60 * 60)] * 40 * 10^6 = (150 * 10^3 * 60 * 60)/(20 * 40 * 10^6) = B.
 
Messages
8,477
Reaction score
34,837
Points
698
Q35)

When the switch S is closed, a current is allowed to flow through the circuit. When the current flows through the circuit, a potential difference is produced across the resistor R. This means that energy is lost in the resistor.

Also, when current goes through the internal resistance of the battery, energy is lost in that internal resistance. Therefore, some of the battery's energy and potential difference is lost in the battery itself.

However, we have to remember the definition of e.m.f - it is the total work done by the battery in driving one coulomb of current around the complete circuit. This work includes the work done by the battery in pushing the current through the internal resistance also, and it is equal to the voltage across the battery when the circuit is not closed (if you put a voltmeter across the battery when the switch S is open, the voltage measured is the rating of the battery AND the e.m.f. value).

Suppose you take internal resistance and separate it from the cells in the battery, then the energy lost in the internal resistance + the energy lost in resistor R is still equal to the e.m.f.

Remember, e.m.f. is the work done by the cells in the battery to push one coulomb through the complete circuit. They will do the same work whether there is internal resistance or not, so the e.m.f. does not change in the circuit because the cells can be assumed to be separate from the internal resistance.

So, A and B are eliminated; e.m.f. does not change at all.

But because a battery = cells + internal resistance, when S is closed, current flows through the circuit and energy is lost in the internal resistance. Also, there is a loss in potential across the internal resistance, so there is a loss in potential across the battery. Therefore, the potential across the battery changes because energy is lost in the internal resistance = C.

Note: if energy is lost in any resistance, it is because work is done against that resistance (suppose you are pushing a box on the ground, friction is the resistance, and you need to do some work against that friction resistance. Your muscles are the potential pushing the current, and some of that energy is lost when you work against friction).
 
Messages
8,477
Reaction score
34,837
Points
698
Q36)

When the switch is open, current goes through both resistors, since there is no other way for it to go. Tab dekhe to we can say that Voltmeter shows reading V (because there is current going through that resistor R2, so there will be potential difference across the resistor too) and there is some reading I on the ammeter, which is equal to (EMF of Source)/(R1 + R2).

(Just imagine the resistors have value R1 and R2).

But if we close the switch, hum dekh sakte hain ki current ko ek aur path nazar ata hai. So, it can go through first resistor, then miss out the second resistor, since current takes path of least resistance.

Therefore, second resistor will have Zero current going through it, so because V = IR, and I = 0, V becomes Zero. So the value of V decreases to Zero.

For I, we can write Current = (EMF of Source)/(R1) since R2 is no longer important in the circuit. Therefore, current increases since resistance of circuit decreases. So current I increases, and V decreases to Zero = B.
 
Messages
8,477
Reaction score
34,837
Points
698
papers.xtremepapers.com/CIE/Cambridge%20International%20A%20and%20AS%20Level/Physics%20(9702)/9702_w09_qp_11.pdf

question 21 the answer is C
can someone explain why?
thank you :)
find the area under the graph. Make it a triangle 1st. A=0.5*100*(2*10^-3)=0.1J but the area is not exactly a triangle. It is slightly more than a triangle so we assume it to be 0.11J
 
Messages
11
Reaction score
29
Points
3
OK here's all of June 2002.

June 2002
==========

1. B

Fact. A would be right if K was given instead of °C.

2. B

You go FORWARD in the direction of X and BACKWARD in the direction of Y.

3. A

The units of speed (msˉ¹)s are equal on both sides.

4. B

You can eliminate A, C and D because they are all nonsense (in my opinion). B is correct because if the timer was started and THEN the ball thrown, you would get 0

height for some time 't' on the x-axis.

5. C

Uncertainity = 2(0.03) + 0.02 = 0.08 = 7%. You multiply the uncertainty of V twice because of the square.

6. D

Air resistance isn't negligible because the speed eventually becomes constant (terminal velocity), so rule out A and C. The Y-axis can't be distance because it

eventually becomes constant, the distance can't become constant during the fall of a body. :p

7. A

Acceleration is constant with uniformly increasing speed. B is increasing rate of speed, C is constant speed and D is at rest.

8. D

S is the distance from cliff to highest point. R is the distance from highest point to sea-level. We are looking for the distance of sea-level to cliff, which is R -

S.

9. B

K.E is ALWAYS conserved in elastic collisions, so K.E before impact is 0.5mv² + 0.5mv² = mv². That means after impact, the K.E should also be mv².

10. B

Fnet = ma.
12 - x = 4 * 0.6
x = 9.6 N.

11. B

Momentum is always conserved so forming an equation,
0 = M1V1 + M2(-V2)
M1V1 = M2V2
V1/V2 = M2/M1

12. D

Fact. Upthrust is very small compared to weight. Also, drag is almost as large as weight not they are not the same (weight is a bit larger) so A and B are wrong.

13. C

Torque = 2 * PD

To find the perpendicular distance, find the perpendicular distance from the force to the pivot and then multiply that by 2. To find the distance from the force to

pivot, construct a triangle and obtain the equation x = 0.15 sin 30.

14. C

Upthrust is the pressure of the block (Pb - Pt) * area, I think.

15. D

Resolve the horizontal 3N and vertical 4N force to get a sideways 5N force which is in the same line as the diagonal 4N force. The resultant force has a magnitude of

1N and the direction is towards the upper-right.

16. D

Efficiency = (useful output)/(total input)

17. C

The only work done is reducing the volume, and since the pressure is constant, work done is p(V1 - V2).

18. B

At Q, the potential energy is 50kJ less than P. This means that the 50kJ must have been converted to kinetic energy. So, K.E (Q) = K.E (P) + 50kJ = 55kJ. And 10 was

lost in friction, leaving us with 45 kJ.

19. D

Simple Power = Force * Velocity.
24 * 10³= 600 * V

20. B

Evaporation occurs over a range of temperatures, while the rest only occur at a fixed temperature.

21. A

Total density = total mass / total volume.

The total mass is m1 + m2 = 2m (since they are equal).

The total volume is MD1/MD2. D1 is ρ and D2 is 2ρ, and that gives us the total volume has 3M/2ρ. Then just use the total density formula I wrote above.

22. C.

Stress = F/A
Strain = extension/length
YM = stress/strain.

23. B

Simple ratio stuff with the YM formula FL/Ax.

24. B

Area below a force/extension graph is energy (i.e. work done). In reducing l2 to l1, this part of the graph is MNQP.

25. C

Speed of electromagnetic waves is always constant, but the frequency will decrease because the wavelength increases.

R M I V U X Y

--> increasing frequency
<-- increasing wavelength

26. B

λ = 4cm.
Time period for one wave is therefore 4 * 0.002s = 0.008s.
F = 1/T = 125 Hz.

27. B

Fact. I α a² and I α 1/r².

28. C

Use the path difference formula (distance from one source - distance from other source) to create an equation involving X. S2x - S1X = λ/2. λ/2 because X is a minimum point.

29. D

λ = 2(1.5) = 3m. 2 nodes are 0.5λ apart.

v = fλ
v = 300(3) = 900.

30. B

Graph X - Diode
Graph Y - Ohmic conductor/metal wire
Graph Z - Lamp

You have to learn these I/V graphs. Also note that a thermistor/semi-conductor has a graph which is like the lamp but has an increasing gradient instead.

31. C

Basic formula recall needed here.

32. A

R = V²/R = 240²/100 = 576. However, this is when the filament is heated so it has increased by 16 times. The normal room temperature would therefore be 576/16 = 36.

33. C

Basic Kirchoff's first law.

34. C

Diagram 2 has the same setup as diagram 1 (2 lamps connected in each parallel setup). If you are confused about diagram 2, then just rotate it. Since they have the same setup, the brightness would obviously be the same.

35. C

The way I solved this was by trying each option until I calculated a 2V drop at R1 and 1V drop at R2. Find the total current using V = IR (where V is 5 and the total resistance is the sum of R1, R2 and R3) and then use the formula again at each resistor to find the voltage drop (5 - V).

36. A

The electron will accelerate towards the +ve plate so A. A is +ve because the field lines are directed from +ve to -ve.

37. A

E = V/d

Increasing the value of 'd' will decrease the value of 'E'. Therefore A is correct.

38. C

Fact.

39. A

Basic stuff. :p

40. C

Work backwards to figure this one out, and see what is happening to the nucleon number and proton number individually. Then # of neutrons is nucleon number - proton number.

November 2002 will probably come next, and then I'll do 2011 backwards.
Can any1 explain me ques 21 :( um confused thnx in advanced
 
Messages
8,477
Reaction score
34,837
Points
698
Can any1 explain me ques 21 :( um confused thnx in advanced
21)
We just take ρ = m / v formula in account to solve this question :) (where ρ is density, m is mass and v is volume)

We are asked to find total density
to calculate total density, we need total mass and total volume
from question we can see that it is given the two masses are equal that means total mass is just sum of m + m = 2m

Now lets look towards total volume so:¬
Masses are equal so I would make two conditions, so that it would be easier to understand (for (i) I would use m and for (ii) I would use m) KEEP Remember masses are equal
i) m is given and we are give it has density ρ so volume of 1st mass = m / ρ
ii) m is given and also given density of 2ρ so volume of second mass = m / 2ρ
add (i) and (ii) it will give us a total volume of 3m / 2ρ

Now, final part:¬
For total total density (td), td = total mass / total volume
=>td = (2m*2ρ) / 3m
=> td = 4mρ / 3m (cut m)​
=> td = 4ρ/3 = A option
:)
 
Last edited:
Top